Line 5: Line 5:
  
 
= [[ECE_PhD_Qualifying_Exams|ECE Ph.D. Qualifying Exam]]: Automatic Control (AC)- Question 1, August 2007=
 
= [[ECE_PhD_Qualifying_Exams|ECE Ph.D. Qualifying Exam]]: Automatic Control (AC)- Question 1, August 2007=
X and Y are iid random variable with
 
 
<math> P(X=i) = P(Y=i) = \frac {1}{2^i}\ ,i = 1,2,3,... </math>
 
 
a) Find <math> P(min(X,Y)=k)\ </math>.
 
 
b) Find <math> P(X=Y)\ </math>.
 
 
c) Find <math> P(Y>X)\ </math>.
 
 
d) Find <math> P(Y=kX)\ </math>.
 
 
----
 
----
=Solution 1 (retrived from [[Automatic_Controls:Linear_Systems_(HKNQE_August_2007)_Problem_1|here]])=
+
==Question==
 
+
Write question here
*To find <math> P(min(X,Y)=k)\ </math>, let  <math> Z = min(X,Y)\ </math>. Then finding the pmf of Z uses the fact that X and Y are iid
+
----
        <math> P(Z=k) = P(X \ge k,Y \ge k) = P(X \ge k)P(Y \ge k) = P(X \ge k)^2 </math>
+
=Solution 1=
 
+
write it here
        <math> P(Z=k) = \left ( \sum_{i=k}^N \frac {1}{2^i} \right )^2 = \left ( \frac {1}{2^k} \right )^2 = \frac {1}{4^k} </math>
+
 
+
*To find <math> P(X=Y)\ </math>, note that  X and Y are iid and summing across all possible i,
+
        <math> P(X=Y) = \sum_{i=1}^\infty P(X=i, Y=i) = \sum_{i=1}^\infty P(X=i)P(Y=i) = \sum_{i=1}^\infty \frac {1}{4^i} = \frac {1}{3} </math>
+
 
+
*To find <math> P(Y>X)\ </math>, again note that X and Y are iid and summing across all possible i,
+
        <math> P(Y>X) = \sum_{i=1}^\infty P(Y>i, X=i) = \sum_{i=1}^\infty P(Y>i)P(x=i)</math>
+
 
+
:Next, find <math> P(Y<i)\ </math>
+
        <math> P(Y>i) = 1 - P(Y \le i) </math>
+
 
+
        <math> P(Y \le i) = \sum_{i=1}^\infty \frac {1}{2^i} = 1 + \frac {1}{2^i} </math>
+
 
+
      <math> \therefore P(Y>i) = \frac {1}{2^i} </math>
+
 
+
:Plugging this result back into the original expression yields
+
        <math> P(Y<X) = \sum_{i=1}^\infty \frac {1}{4^i} = \frac {1}{3} </math>
+
 
+
 
+
*To find <math> P(Y=kX)\ </math>, note that X and Y are iid and summing over all possible combinations one arrives at
+
        <math> P(Y=kX) = \sum_{i=1}^\infty i = 1^\infty P(Y=ki, X=i) = \sum_{i=1}^\infty P(Y=ki)P(X=i) </math>
+
:Thus,
+
        <math> P(Y=kX) = \sum_{i=1}^\infty \frac {1}{2^{ki}} \frac {1}{2^i} = \sum_{i=1}^\infty \frac {1}{2^{(k+1)i}} = \frac {1}{2^{(k+1)}-1} </math>
+
 
----
 
----
 
==Solution 2==
 
==Solution 2==

Revision as of 08:07, 27 June 2012


ECE Ph.D. Qualifying Exam: Automatic Control (AC)- Question 1, August 2007


Question

Write question here


Solution 1

write it here


Solution 2

Write it here.


Back to ECE Qualifying Exams (QE) page

Alumni Liaison

Questions/answers with a recent ECE grad

Ryne Rayburn